4
$\begingroup$

If I had a function $f$ that was this: $$f\left(a_{0},a_{1},a_{2},a_{3},...a_{\infty},b_{0},b_{1},b_{2},b_{3},...b_{\infty}\right)=\frac{\sum_{k=0}^{\infty}\frac{1}{k+1}\left(\sum_{l=0}^{k}b_{l}\left(k-l+1\right)a_{k-l+1}\right)}{\int_{0}^{1}\sqrt{\left(\sum_{n=0}^{\infty}na_{n}t^{n-1}\right)^{2}+\left(\sum_{n=0}^{\infty}nb_{n}t^{n-1}\right)^{2}}dt}$$ What combination of $a_{0},a_{1},a_{2},a_{3},...a_{\infty},b_{0},b_{1},b_{2},b_{3},...b_{\infty}$ would get me the maximum value of the function? $$$$ Context: I've derived this problem from making a parametric function that has ordinary power series for both x and y coordinates. I then used Green's theorem to make a general formula for the area of the parametric function and put it on top. I used the arc length formula on the function and put it at the bottom. This should mean that if I am able to maximize the function, I will be able to discover the shape that takes up the most area for perimeter (which should be the circle). This came from a proof I saw somewhere where someone proved that the line is the most efficient way to travel between two points using calculus. So, I thought I could try and do the same here, but with a different problem. Overall, the problem had become pretty challenging and more difficult than I had previously imagined, and I am attempting to do some work to simplify both the numerator and the denominator into ordinary power series, which I then can try to maximize the numerator and minimize the denominator, or something similar to that. Unfortunately, I have no idea how to use gradient descent on a function with infinitely many parameters, which is why I've come to you guys to see if I can get any help. I'll give updates if I discover something new that could help. $$$$ Update 1: $f\left(a_{0},a_{1},a_{2},a_{3},...a_{\infty},b_{0},b_{1},b_{2},b_{3},...b_{\infty}\right)=\frac{\sum_{k=0}^{\infty}\frac{1}{k+1}\left(\sum_{l=0}^{k}b_{l}\left(k-l+1\right)a_{k-l+1}\right)}{\sum_{k=0}^{\infty}\frac{p_{k}}{k+1}}$, where $p_{k}=\frac{\sum_{l=0}^{k+2}l\left(k-l+2\right)\left(b_{l}b_{k-l+2}+a_{l}a_{k-l+2}\right)-\sum_{l=1}^{k-1}p_{l}p_{k-l}}{2\sqrt{\left(b_{1}\right)^{2}+\left(a_{1}\right)^{2}}}$ is an equivalent form of the function. However, this new form comes with cons, such as being recursive, so I can't really do much with it yet. I did this by expanding the bottom and solving for a general formula to square root a power series, and then plugging the bottom one in and integrating.

$\endgroup$
5
  • 7
    $\begingroup$ The wording of this question makes it appear that you are assigning us homework. You are not in a position to do that. $\endgroup$ Commented Nov 28 at 3:51
  • 5
    $\begingroup$ Sorry about the strange wording of the question; it is not homework. This comes from me trying to prove that the circle is the shape that takes up the most area in relation to its perimeter using calculus. I just thought wording it that way would make it clear what the primary objectives of the problem were. $\endgroup$ Commented Nov 28 at 3:55
  • 4
    $\begingroup$ OK, that's better – BUT users here are expected to provide some context for their questions. Context is a tricky word, but it can mean give some motivation for how you came across this question, why you think people should be interested in it, what progress you've made on it, where you got stuck, what mathematical tools you have at your disposal (so we don't write you a solution using stuff you've never heard of), etc. See math.meta.stackexchange.com/questions/9959/… $\endgroup$ Commented Nov 28 at 4:25
  • 4
    $\begingroup$ @GerryMyerson This post is also helpful math.meta.stackexchange.com/a/39604/195378 . Currently, the question has already 4 closing votes. I hope it won't be closed :) $\endgroup$ Commented Nov 28 at 9:13
  • 3
    $\begingroup$ Thank you all for your help in refining this question. I've added some additional context and motivation for the problem, which I think should be enough to reopen the question. $\endgroup$ Commented 2 days ago

0

You must log in to answer this question.